subject
Mathematics, 01.10.2019 13:00 sarak2212

If m < 1 = 80, then m < 2 is:

if m < 2 = 63, then m < 3 is:


If m &lt; 1 = 80, then m &lt; 2 is:  if m &lt; 2 = 63, then m &lt; 3 is:

ansver
Answers: 1

Another question on Mathematics

question
Mathematics, 21.06.2019 16:30
Ineed if you could explain and give me the answer you! this needs done
Answers: 1
question
Mathematics, 21.06.2019 20:00
15 and 14.7 are 1 apart, so 15 – 14.7 must be 1.
Answers: 1
question
Mathematics, 21.06.2019 23:00
What is the circumference of the pie
Answers: 2
question
Mathematics, 22.06.2019 01:00
The table shown below gives the approximate enrollment at the university of michigan every fifty years. how many more students were enrolled at the university of michigan in 1950 than in 1900?
Answers: 3
You know the right answer?
If m < 1 = 80, then m < 2 is:

if m < 2 = 63, then m < 3 is:
...
Questions
question
Mathematics, 09.03.2021 04:00
question
English, 09.03.2021 04:00
question
English, 09.03.2021 04:00
question
Mathematics, 09.03.2021 04:00